Civ Pro term 2 MCs

Réussis tes devoirs et examens dès maintenant avec Quizwiz!

A plaintiff sued a defendant for trespass. The defendant claimed the property in question belongs to her, not to the plaintiff. After a lengthy search through old family records stored in the defendant's attic, her attorney found an old deed that appears to provide a missing link in the plaintiff's title. The attorney immediately placed the deed in his office safe as part of his working file on the defendant's case. The plaintiff learned of the existence of the deed through other sources and seeks to inspect it and copy it. Under modern discovery rules, may the plaintiff do so? A. Yes, because the deed was not prepared in anticipation of litigation. B. Yes, because the discovery rules trump the attorney work product doctrine. C. No, because the efforts of the defendant's attorney constitute attorney work product. D. No, because the deed was obviously never officially recorded and is therefore irrelevant to the case.

A

A worker filed a wrongful termination claim against his employer in federal court. The employer moved to dismiss for lack of personal jurisdiction, which the court denied. The employer then filed an answer. The day after filing and serving the answer, the employer filed a motion for judgment on the pleadings, raising the defense of failure to state a claim. Has the employer waived this defense? A. No, because a party may raise the defense of failure to state a claim anytime through trial. B. No, because twenty-one days have not yet passed from the time the answer was served. C. Yes, because a motion under Rule 12 must be made before filing a responsive pleading. D. Yes, because Rule 12 defenses must be consolidated or they are waived.

A

On June 1 a customer filed a complaint in federal court against a State A shopkeeper alleging the following: "On April 2, 2014 the customer slipped on a puddle of water located in an aisle of the store owned by the shopkeeper, severely injuring himself as a result. Therefore, the customer is entitled to $100,000 to compensate for his medical expenses, pain, and suffering." To be liable for personal injuries occurring on one's premises under the applicable State A law, the owner of the property (or its employees) must have been negligent in creating or failing to eliminate the dangerous condition or must have been aware of the dangerous condition and failed to warn invitees of the condition. On June 20 of the same year the shopkeeper filed an answer to the complaint denying all of the material allegations. One week later, the shopkeeper filed a motion for judgment on the pleadings based on the customer's failure to state a claim. How is the court likely to rule on this motion? A. grant the motion, bc the customer has failed to allege facts that, if true, show that the shopkeeper was negligent or breached its duty to warn B. grant the motion, bc the customer has not offered any evidence proving that the property owners are responsible for the dangerous condition that harmed the customer C. deny the motion, bc the customer has pleaded facts that, if true, show plausible entitlement to relief D. deny the motion, bc the shopkeeper was required to file the motion to dismiss prior to filing an answer to the complaint

A

An employee from State A initiated an action in State A federal court against its employer (a State B corporation headquartered in State B) for employment discrimination (based on sexual harassment occurring in State A), seeking $1,000,000 in damages. The employee served process by personally delivering the summons and complaint herself to the CEO of the employer (the employee did not use a process server or other agent to achieve service). This was done on June 15. On June 30 the employer filed a motion to dismiss for lack of personal jurisdiction, which the court denied on July 5. Having heard no response from the employer, on September 1 of the same year the employee sought and obtained an entry of default. The employee then applied for and obtained a final default judgment from the court (the employer was duly notified of the hearing on the application for default judgment but did not appear). Twenty-nine days after entry of the default judgment, the employer filed a motion for relief from the judgment. Which of the following would be an appropriate response of the court to this motion? A. Deny the motion, because the employer failed to move for relief from judgment within a reasonable period of time. B. Deny the motion, because the employer failed to move for relief from judgment within a reasonable period of time. C. Grant the motion, because parties are not permitted to serve process themselves, meaning the employer was not properly served with process under Rule 4. D. Grant the motion, because the court lacked personal jurisdiction over this dispute.

B

On July 1 the owner of a pizza parlor brought an action in federal court against a former employee for embezzlement of $80,000. The former employee was served with the complaint on July 15 that same year and responded with an answer that was served on the owner on the following August 5. Two days later, the owner filed an amended complaint, seeking $100,000 in damages. On August 25 the owner filed a second amended complaint, adding a conversion claim. Is the owner's second amended complaint permissible as a matter of course? A. no, bc more than 21 days have passed since the complaint was served B. no, bc the owner previously amended the complaint C. yes, bc the complaint may be amended as a matter of course w/in 21 days of service of the former employer's answer D yes, bc amendments are to be freely granted when justice so requires

B

You are an in-house counsel for an insurance company. One of its life insurance policyholders has recently passed away and each of his three children claims the right to the proceeds from the policy. Which type of action would you bring on the insurance company's behalf? A. An impleader action. B. An interpleader action. C. An action for an injunction. D. A motion to dismiss.

B

A citizen initiated an action against a police officer asserting a claim under 42 U.S.C. § 1983. During the course of a deposition of the police officer the attorney for the citizen asked, "Describe what happened when you had the plaintiff in custody." The attorney for the police officer objected, saying "Objection; the police officer cannot possibly remember what happened so long ago." The police officer then proceeded to answer the question by saying, "I can't recall what occurred during that period." You are the attorney for the citizen. Is there anything improper that you can identify in the exchange above? A. As a defendant, the police officer may not be compelled to testify against himself. B. The objection by the police officer's attorney is improper because it is suggestive. C. The police officer's answer is improper because it is nonresponsive. D. The question of the citizen's attorney is improper because it is too vague.

B The police officer's attorney is suggesting an answer and the Federal Rules prohibit objections that suggest how the deponent should respond. Such objections are called speaking objections. Fed. R. Civ. P. 30(c)(2). See, e.g., Specht v. Google, Inc., 268 F.R.D. 596 (N.D. Ill. 2010) ("Objections that are argumentative or that suggest an answer to a witness are called "speaking objections" and are improper under Rule 30(c)(2).").

The State A promoter of a sporting event had a contract with ABC Clothing (a State B corporation) to supply all the shirts for people staffing the event. However, the promoter subsequently signed another agreement with a competitor of the clothing company, XYZ Clothing (a State B corporation), to supply shirts instead. ABC initiated an action in federal court against the promoter for breach of contract, seeking to enjoin the promoter from proceeding under the contract with XYZ and requesting specific performance of its original agreement. The promoter filed a motion to dismiss for failure to join XYZ as a required party defendant in the action. How should the court rule on the motion? A. Deny it, because XYZ's presence in the action would destroy complete diversity, making its joinder not feasible. B. Deny it, because the dispute between the promoter and ABC can be resolved completely without the presence of XYZ and no other grounds for requiring the joinder of XYZ exist. C. Grant it, because if XYZ is not joined in the action it may later file a claim against the promoter, thereby subjecting the promoter to a substantial risk of incurring inconsistent obligations. D. Grant it, because XYZ's interest in supplying shirts for the event will be impaired by a resolution of the dispute between ABC and the promoter.

B The court can grant complete relief to ABC in XYZ's absence; with the promoter before the court, the court will be able to order it to comply with its contract with ABC and refrain from proceeding under its contract with XYZ. None of the other requirements are satisfied because if the promoter is under an injunction to perform under its contract with ABC and to refrain from performing under its contract with XYZ, XYZ may simply sue the promoter for a breach of its contract, permitting XYZ to protect its interests. The promoter could comply both with an injunction in the action by ABC and with any order awarding damages to XYZ in a separate lawsuit. Fed. R. Civ. P. 19(a); see also MasterCard Int'l v. Visa Intl. Serv. Ass'n, 471 F.3d 377 (2d Cir. 2006).

A State A trust company was in possession of $70,000 in cash that had been given to it by the settlor of a trust. The settlor died, leaving three survivors (two from State A and another from State B) who each now claim entitlement to the entirety of the trust proceeds as trust beneficiaries. The two State A survivors have each sent letters to the trust company demanding immediate full payment of the $70,000; the State B survivor has sent a similar letter to the trust company. You represent the trust company as its attorney. Is it possible to initiate an action in federal court that will permit the trust company to handle this situation? A. No, because there is not complete diversity between the State A trust company and the claimants, two of whom are also from State A. B. No, because the amount in controversy is less than the required amount. C. Yes, because there is minimal diversity among the claimants and a sufficient amount in controversy. D. Yes, because the trust company can assert a declaratory judgment action against the State B survivor based on diversity jurisdiction, at which point the State A survivors may be impleaded as third-party defendants.

C

A chef sued a restaurant in state court for tortious interference with prospective business advantage. Six days after receiving service of process, the restaurant removed the case to federal court. Fourteen days after the notice of removal was filed the chef sought entry of default in federal court, which the clerk entered. You represent the restaurant. After receiving notice that a hearing on the chef's application for default judgment will occur in 14 days, what is the best way to respond to this development? A. Appear at the hearing on the application for default judgment to oppose it. B. File an answer to the complaint. C. Move the court to set aside the entry of default as improperly granted. D. Move the court to set aside the entry of default based on excusable neglect.

C

A pedestrian from State A brought a negligence action in federal court against a State B motorcyclist and a State C truck driver, seeking $100,000 from each for injuries arising out of an accident. The motorcyclist responded by asserting a $50,000 claim for negligence against the truck driver. Is the motorcyclist's claim properly joined? A. no, bc the negligence claim does not contend that the truck driver is liable to the motorcyclist for all or part of the pedestrians claim against the motorcyclist B. no, bc the amount in controversy is insufficient C. yes, bc the claim arises out of same occurrence that is subject matter of pedestrians claim D. yes, bc the motorcyclist and truck driver are completely diverse

C Joinder of the claim is permissible under Rule 13(g) as a crossclaim because the claim arises out of the same accident that is the basis for the pedestrian's claim against the motorcyclist. Further, although the amount in controversy is insufficient, there would be supplemental jurisdiction over the motorcyclist's claim under 28 U.S.C. § 1367

During the course of a medical malpractice action between a doctor and a patient, the hospital where the disputed medical care was given was served with a subpoena asking to depose the CEO of the hospital and for the CEO to produce "all documents pertaining to the medical care given to the patient during the visit at issue in this case; any such documents should be converted to an electronic format." Which of the following would be an appropriate objection for the hospital to raise in response to this request? A. The request improperly asks for documents pertaining to the patient's physical condition, which is not in controversy in this case. B. The request improperly asks for documents that are protected by the doctor-patient privilege. C. The request improperly asks for documents to be converted to an electronic format. D. The request improperly asks for documents to be produced by a nonparty to the action.

C Producing parties are not obligated to convert documents into an electronic format, even if the requesting party specifies such conversion in its request. Rule 45 permits producing parties to produce material in the form that it is ordinarily maintained. Fed. R. Civ. P. 45(e)(1)(A).

A baker initiated a breach of contract action in federal court against a flour supplier for providing poor quality flour over the past three years. During discovery, the baker issued a request for "All records pertaining to the production, purchase, and maintenance of flour and sugar over the past three years." The supplier objected and produced records relating only to flour over the past three years. If the baker were to move for an order compelling the supplier to produce the records pertaining to sugar over the past three years (after meeting and conferring with the baker), how would the court be likely to rule? A. Deny the motion, because the requested documents are outside the scope of discovery. B. Deny the motion, because producing such records would be unduly burdensome. C. Grant the motion, because the records are relevant to the baker's claim in the action. D. Grant the motion, because the records relate to the subject matter of this action.

A

A bank made a loan to a borrower at an annual interest rate of 40%. A different bank also made a loan to the same borrower at an annual interest rate of 40%. After viewing a television program about loan sharks and illegal rates of interest, the borrower initiated a declaratory judgment action against both banks in a single action in federal court seeking rescission of the loan agreements on the ground that they were usurious. The banks objected to being joined in a single action in this fashion. Is such joinder permissible under the Federal Rules? A. no, bc the claims being asserted against the banks do not arise from the same transaction, occurrence or series of transactions or occurrences B. no, bc there is no question of law or fact common to the defendants C. yes, bc the issue of usury is a common question of law among the claims against the banks D. yes, bc the joinder is necessary to accord complete relief to the borrower

A

A bicyclist initiated an action in State A federal court against the driver of a car that hit the bicyclist in State A, causing severe injuries. The driver's attorney filed an answer asserting the defense of contributory negligence, identifying conduct of the bicyclist that contributed to the collision. The attorney for the bicyclist responded by serving a motion for sanctions on the driver, indicating that State A law fails to recognize contributory negligence as an affirmative defense. One month later, because the driver had not withdrawn the answer containing the contributory negligence defense the attorney for the bicyclist filed the motion for sanctions with the court. After a hearing on the motion, the court entered monetary sanctions against the driver in the amount of $10,000. Was the court's sanctions award appropriate? A. no, bc a court may not impose monetary sanctions on a represented party for asserting frivolous legal arguments B. no, bc the amount of sanctions exceeds the permissible amount C. yes, bc the driver raised a defense that was not supported by existing law or based on a non frivolous argument for modifying the law D. yes, bc the bicyclist served the motion on the driver 21 days before filing the motion with the court

A

A citizen of State A was arrested by a police officer from the State A police department. After realizing that he was mistaken and had the wrong person, the officer released the citizen. However, the officer made a note of the citizen's identity and entered it into a database that was a "watch list" containing the names of suspicious persons. If an officer from any jurisdiction encountered a person on this list, the officer was empowered to detain the person overnight and question them. One year after the initial arrest, the citizen was pulled over for a traffic stop by a State B police officer. When the State B police officer discovered that the citizen was on the aforementioned watch list, the officer arrested the State A citizen and detained him at the police station overnight. The State A citizen initiated an action in State A federal court against the State A police officer and the State B police officer, seeking monetary damages for the violation of the citizen's civil rights. The State A police officer moved to sever the joined claims. How should the court rule on the motion? A. deny the motion, bc the claims against the State O officer and State B police officer share a common question of law and arise out of same series of occurrences B. deny the motion, bc the State A police officer is a necessary party in the case C. grant the motion, bc the claims against the State A officer and State B police officer do not share a common question of law D. grant the motion, bc the claims against the State A officer and State B police officer do not arise out of the same transaction, occurrence, or series of transactions or occurrences C. D.

A

A customer choked on a bone in her fish fillet while eating at a restaurant. The restaurant's manager, who was not in the restaurant when the incident occurred, met with the customer and the restaurant employees who witnessed the incident. As required by state law, the manager prepared a report regarding the incident and filed it with the state board of health. The manager then accepted a job in another country and moved there. The customer subsequently commenced a civil action against the restaurant in a federal court, alleging negligence and a breach of the warranty of merchantability. The customer's attorney served a request for all documents and reports prepared by the restaurant relating to the incident. Must the restaurant produce the manager's report? A. Yes, because the manager prepared the report to comply with state law rather than to prepare for trial. B. Yes, because the customer can establish that she has a substantial need for the report to prepare her case. C. No, because the manager's report is hearsay and therefore not discoverable. D. No, because the report constitutes work product and is therefore not discoverable.

A

A homeowner from State A initiated an action in State A federal court against a homebuilder—an individual from State B—for breach of warranty for faulty construction of the home, seeking $100,000. The homebuilder was served with process in State A on July 1, 2017. By September 1, 2017, there had been no response from the homebuilder. Thus, the homeowner sought and obtained entry of default from the clerk on September 5, 2017. The homeowner applied for a default judgment, and the hearing on the application was set for October 1. The homebuilder received notice of the default judgment motion and hearing on September 30, 2017. At the hearing—at which the homebuilder did not appear—the court heard the homeowner's evidence establishing the amount of damages. Ultimately, total damages from the faulty construction amounted to $108,345.50; thus, the court entered a default judgment for that amount. Two days later, the homebuilder moved to set aside the default judgment. Which of the following represents the homebuilder's best argument in favor of setting aside the default judgment? A. the court awarded damages to the homeowner that exceeded the amount demanded in the initial complaint B. the court lacked personal jurisdiction over the homebuilder, making the default judgment void C. the homebuilder received insufficient notice of the hearing on the application for default judgment D. the homeowner waived the right to pursue a default judgment by seeking default more than 30 days after the homebuilder defaulted

A

A manufacturer filed a federal action against a competitor asserting a claim for tortious interference with prospective economic advantage. The basis for the manufacturer's claim was that the competitor had contacted a prospective client of the manufacturer and offered to sell a similar product for a lower price. Assume that under the relevant law, this type of conduct would be insufficient to establish tortious interference with prospective economic advantage. Upon reviewing the complaint, the court issued an order to the manufacturer's attorney to show cause why the attorney should not be sanctioned for filing a complaint for tortious interference with prospective economic advantage based on these facts. After receiving briefs on the matter from both sides, the court issued an order imposing sanctions on the manufacturer's attorney directing payment to the competitor of the attorney's fees it had incurred in the action up to that point. Is the court's order for sanctions appropriate? A. no, a court may only order the payment of attorney's fees under these circumstances in response to a motion for sanctions, not on its own initiative B. no, bc the court did not hold a live hearing on the matter prior to imposing sanctions C. yes bc the manufacturers claim is not supported by existing law or a non frivolous argument for modifying existing law D. yes, bc the award of attorney's fees is warranted to achieve effective deterrence of the manufacturer's conduct

A

A passenger in a car initiated an action in federal court against the driver of the car based on the driver's negligence in causing a collision. Twenty days later, the driver simultaneously filed an answer to the passenger's complaint and a third-party claim against a pedestrian, claiming that the pedestrian was liable to the driver for recklessness based on the pedestrian's jaywalking that led to the accident. Is the driver's third-party claim properly joined? A. no, bc the claim does not assert that the pedestrian is liable to the driver for all or part of passengers claim against the driver B. no, bc the driver did not seek and obtain courts permission before asserting third party claim C. yes, bc the claim arises out of same occurrence that is basis for passengers claim against the driver D. yes, bc there will be supplemental jurisdiction over third party claim

A

A patent holder initiated a patent infringement action against a cellphone company based on its production of a smartphone device. In the course of discovery, the patent holder requested "All documents relating to the design of the smartphone device." The company responded by producing the requested documents. The patent holder responded with an interrogatory asking for "The names of the persons or entities who either supplied or participated in supplying documents that were produced in response to the previous request for production." The company objected to this request. If the company ultimately moves for a protective order (after meeting and conferring with the patent holder), how is the court likely to rule? A. Deny the motion, because such information is likely to lead to the discovery of admissible evidence. B. Deny the motion, because protective orders are not available in connection with interrogatories. C. Grant the motion, because such "discovery about discovery" is not relevant to a claim or defense raised in the action. D. Grant the motion, because such information is protected by the work product doctrine.

A

A pesticide manufacturer (incorporated in State A) was dumping its waste products into a river adjacent to its plant. Ten State A homeowners whose houses also abutted the river were experiencing several problems from the waste that had been dumped, including experiencing a foul odor and suffering from headaches. The homeowners wish to assert claims for nuisance and violation of state and federal environmental protection statutes as a class, seeking an injunction ordering the manufacturer to cease the dumping of its waste, compensatory damages for any loss to their property values, and medical monitoring to determine if there have been any long-term adverse health consequences from their exposure to the waste. Is this group likely to be certified as a class? A. No, because the group is not sufficiently numerous to make joinder of all members impracticable. B. No, because the group seeks injunctive and monetary relief in the same action, the latter of which will require individualized determinations of harm and valuation. C. Yes, because there is federal question jurisdiction over the federal environmental statute claim and supplemental jurisdiction over the remaining claims. D. Yes, because the homeowners' claims share at least one common question of law—the legality of the manufacturer's dumping of waste into the river.

A

A plaintiff is suing a defendant in federal court for personal injuries arising out of an automobile accident. The defendant's automobile insurance policy covers the accident. Must the defendant disclose the existence of the insurance policy? A. Yes, because it is required under the Federal Rules. B. Yes, unless the discovery of the insurance coverage would not lead to other discoverable evidence. C. No, unless the plaintiff submits an interrogatory. D. No, because insurance coverage cannot be mentioned at trial.

A

A shareholder bought 1,000 shares of a global energy company on August 20. At the time, there were 100,000 shares of the company's stock outstanding. Unfortunately for the shareholder, on August 21 it came to light that the company's management was artificially inflating the company's earnings the past year in order to inflate the company's stock price. By August 27, the stock was worthless. It is unquestioned that the shareholders who owned the stock during the week of August 21-27 were harmed. The shareholder filed a class action lawsuit in the federal district court. Must she give notice to other absent class members? A. Yes, because the class action is a "common question" class action. B. Yes, because due process always requires notice to absent class members. C. No, because notice would be impracticable under these circumstances. D. No, because notice to absent class members is never required.

A

A state's civil procedure rules allow for "nail and mail" service (posting the summons at the defendant's dwelling and thereafter mailing him a copy by certified mail) when "regular" service (service by physical delivery or by leaving a copy of the complaint with someone of suitable age and discretion at the defendant's usual place of abode) cannot be accomplished with due diligence. A plaintiff brought suit against a defendant in the federal district court for that state. After the plaintiff's special process server made many attempts at serving the defendant, the plaintiff's attorney directed the process server to nail the complaint and summons to the defendant's front door, and the attorney mailed (by certified mail) a copy to the defendant in accordance with the state rule. Has the defendant been properly served? A. Yes, if "nail and mail" service is reasonably calculated to give the defendant notice of the action. B. Yes, because under the Erie doctrine the federal court must apply the state's service of process rules. C. No, if the defendant only rented the dwelling at which service was posted. D. No, because "nail and mail" service is not authorized by the Federal Rules of Civil Procedure.

A

Google and Microsoft (both State A corporations headquartered in State A) collaborated to produce a phone. Unfortunately, the phone exploded when a State B consumer was using it. The consumer initiated a federal diversity action against Microsoft, seeking damages on a products liability claim. Microsoft responded by impleading Google into the case, asserting that Google was liable to the consumer for the defect, not Microsoft. Is Microsoft's joinder of Google proper? A. No, because Microsoft is not asserting a claim against Google contending that Google is liable to Microsoft for all or part of the liability that Microsoft may have to the consumer. B. No, because Microsoft and Google are not diverse. C. Yes, because Microsoft is alleging that Google is liable for all of the harm caused to the consumer. D. Yes, because the claim arises out of the same occurrence that is the basis for the consumer's claim against Microsoft.

A

In a breach of contract action by ABC, Inc. against XYZ Corp., XYZ sought copies of all correspondence relating to the contract at issue in the dispute. ABC responded by producing some documents and withholding others based on the attorney-client privilege. It disclosed information about what it was withholding by producing a privilege log. Attorneys for XYZ determined that one document claimed as privileged by ABC should not be regarded as privileged. After meeting and conferring with ABC, XYZ moved for an order compelling ABC to produce the document in question. The court agreed with XYZ that the document was not privileged and ordered ABC to produce it within one week. Two weeks later, ABC had still not produced the document at issue. After this failure came to the court's attention in a letter to the court from XYZ, the court entered an order to show cause why ABC should not be sanctioned. After hearing no response, the court directed that the facts alleged by XYZ in the case on the issue covered by the unproduced document would be taken as established for the purpose of this case. Was the sanction order an abuse of discretion under these circumstances? A. No, because ABC failed to comply with the court's discovery order and the sanction is authorized by the Federal Rules. B. No, because the court has inherent authority to sanction the parties before it. C. Yes, because a court does not have the authority to impose sanctions under these circumstances absent evidence of bad faith. D. Yes, because a court does not have the authority to impose sanctions for discovery violations absent a motion for sanctions.

A

On June 1 a patient filed and served a complaint in federal court against a surgeon for medical malpractice based on alleged mistakes the surgeon made during a recent surgery. After the relevant statute of limitations period expired on June 30 of that same year, the patient filed an amended complaint seeking to add the hospital where the surgery occurred as a defendant. The hospital objected to the amendment on the ground that it sought to assert a claim against it that was untimely. Will the amended complaint relate back to the time of filing, thereby defeating the untimeliness challenge? A. no, bc there are no facts indicating that the failure to name the hospital as a defendant originally was due to a mistake concerning the proper party's identity B. no, bc the patient either know or should have known that the hospital should have been a party in the case C. yes, bc the newly asserted claim arises from the same occurrence that is the basis for the patient's original claim D. yes, bc the hospital received notice of the action within 90 days of the filing of the original complaint

A

On March 1 an actor filed a defamation action against a newspaper in State A federal court; service of process was achieved the same day. Twenty days later the newspaper responded with a motion to dismiss for failure to state a claim, attaching an affidavit of a witness who attested to the truth of the information that appeared in the news story about the actor at issue in the case. This motion was served on the actor and the court took the motion and affidavit under advisement. Instead of responding to the motion, one week later the actor filed a notice of dismissal. The newspaper objected to the notice of dismissal and asked the court to strike it. Is the actor's notice of dismissal permissible? A. No, because the newspaper's motion is treated as a summary judgment motion, which prevents the actor from voluntarily dismissing his claim. B. No, because the actor filed the notice more than 21 days after he filed his complaint. C. Yes, because the newspaper has yet to serve an answer or a motion for summary judgment. D. Yes, because this is the actor's first notice of dismissal.

A

Seven employees of a bank initiated an action in federal court against the bank alleging employment discrimination. Each of these employees asserted that they were passed over for promotions because of their age, although each employee worked in different departments under different supervisors. The employees further alleged that this discriminatory treatment was pursuant to the bank's policy of preferring younger employees. The bank moved to sever the claims of the employees, arguing that their joinder was not permitted under the Federal Rules. How is the court likely to rule? A. deny the motion, bc the employees are each victims of the same policy and the ct will need to determine whether that policy was discriminatory for each of the claims B. deny the motion, severance is not a remedy for misjoinder C. grant the motion, bc the alleged discrimination occurred at the hands of different supervisors in different departments, which means that no common question of law or fact exists D. grant the motion, bc the employees are not sufficiently numerous to warrant certifying the group as a class

A

While crossing the street, a pedestrian was struck by a motorcyclist. The pedestrian filed a negligence complaint against the motorcyclist, which read, "I was stuck by the motorcyclist while I was crossing the street. I wanted to see what it was like to be blind so I closed my eyes and crossed once I thought the sign said I could walk. Unfortunately, the sign actually read, 'Don't walk' and I was hit. I demand $100,000 in damages." Under the applicable law, crossing a street with one's eyes closed while the sign reads "Don't walk" would constitute contributory negligence, an affirmative defense for the defendant that would completely bar any relief for the pedestrian. The motorcyclist has just received this complaint and needs to form a response. Which of the following is the best response that the motorcyclist can make to this complaint? A. Move to dismiss for failure to state a claim, because the complaint is self-defeating. B. Move for a judgment on the pleadings, asserting the affirmative defense as a ground for judgment in favor of the defendant. C. Move for sanctions, because the pedestrian's claim is not warranted under the law. D. File an answer in which the affirmative defense of contributory negligence is raised.

A

A former employee initiated a breach of contract claim against an employer for terminating the employee prior to the end of the contract period without paying the agreed upon severance payment to the employee. During discovery, the employer moved for an order requiring the former employee to submit to a mental examination by an independent physician. The employer intends to show that the former employee's psychological problems motivated the termination. Which of the following would be the best ground for opposing this request? A. The employer cannot show good cause for obtaining a mental examination. B. The exam would be outside the scope of discovery. C. The exam would be unduly burdensome. D. The mental condition of the former employee has not been put in controversy by the former employee.

A A breach of contract dispute over an unpaid severance does not reasonably present (or at least the facts do not suggest it presents) any real need for mental or psychological evaluations. Such a dispute can be resolved based on the terms of the contract and whether the conditions triggering the obligation to pay a severance were or were not satisfied. There is no indication that the psychological problems of the employee bear on that issue in a way that would justify the intrusion accompanying a mental examination. Thus, the employer cannot show that the mental condition of the former employee is genuinely at issue nor can it establish the "good cause" required under the Rule. Fed. R. Civ. P. 35(a)(2)(A). See also Schlagenhauf v. Holder, 379 U.S. 104, 119-120 (1964) ("Here, however, Schlagenhauf did not assert his mental or physical condition either in support of or in defense of a claim. His condition was sought to be placed in issue by other parties. Thus, under the principles discussed above, Rule 35 required that these parties make an affirmative showing that petitioner's mental or physical condition was in controversy and that there was good cause for the examinations requested.").

A customer initiated an antitrust action against ABC, Inc. in federal court, seeking $500,000 in damages. During discovery the customer requested "All documents relating to discussions with competitors about market share and pricing over the last 20 years." ABC informed you, ABC's lawyer, that such documents beyond the most recent 10 years were located exclusively on backup tapes and that it would cost $100,000 to extract the requested documents from them, an amount ABC does not want to pay. Which of the following would be the most effective response to the customer's request in light of the information ABC has given to you? A. Decline to produce the documents located on the backup tapes by identifying them as not reasonably accessible. B. Move for a protective order from the court permitting ABC to withhold the documents. C. Produce the backup tapes to the customer and permit the customer to inspect them at its own expense. D. Retrieve a sample set of documents from the backup tapes to evaluate their relevance to the plaintiff's claims.

A The Federal Rules permit producing parties to withhold inaccessible ESI (electronically stored information) so long as they identify it as not reasonably accessible. Fed. R. Civ. P. 26(b)(2)(B). This is the most effective response because if successful, it would result in ABC not having to incur any expense associated with responding to the request for these documents

A car collector bought a car with gold leaf paint from a manufacturer. During the first month, all the gold leaf paint peeled off. The collector sued the manufacturer, and during discovery served an interrogatory asking the manufacturer to identify all other purchasers of the gold leaf paint over the previous 10 years. The manufacturer was aware that only about 25 of the 2 million buyers of its cars have ordered the gold leaf option. The manufacturer has retained copies of all sales forms, but has not maintained separate files of the buyers of each particular option. In a court using the federal rules, what are the manufacturer's obligations with respect to the collector's interrogatory? A. It must search its files and then disclose the information. B. It may allow the collector to search the records himself. C. It may respond by stating that only about 25 of the 2 million buyers ordered the option. D. It may state that searching the records is too great a burden and so it is excused from answering the interrogatory.

B

A doctor from State A injured a patient from State B in the course of performing a surgery. On the day before the statute of limitations period expired, the patient initiated a $100,000 action for medical malpractice against the doctor in the U.S. District Court for the District of State A. The two days after that case was filed, a different patient from State C filed a malpractice action against the same doctor in the same State A federal court, seeking $1 million in damages. In both cases, the plaintiffs alleged that the doctor had used a machine that was not certified for use in the type of surgeries that both patients had undergone. You represent the patient from State B. Is there a way to proceed if you want to join with the State C patient to assert the State B patient's claim? A. No, because the two claims do not arise out of the same occurrence. B. Yes, by moving to consolidate the actions. C. Yes, by voluntarily dismissing the State B patient's claim and moving to intervene in the State C plaintiff's action. D. Yes, by voluntarily dismissing the State B patient's claim and having the State C patient amend her complaint to name the State A patient as a co-plaintiff.

B

A farmer from State A visited an attorney to seek assistance with asserting a claim. The farmer told the attorney that his neighbor from State B had trespassed on his property by walking across it (without causing any damage) and thus the farmer wanted to sue the neighbor. In State A, the essential elements of an action for trespass are: (1) ownership or possessory interest in land (2) invasion, intrusion or entry by the defendant affecting the plaintiff's exclusive possessory interest; (3) done intentionally; and (4) causing direct injury. Based on the information received from the farmer, the attorney initiated an action for trespass in federal court against the neighbor alleging, "The neighbor trespassed on the farmer's property, wherefore the farmer demands damages in the amount of $100,000." When the neighbor received service of process in the action, the neighbor served on the farmer a motion for sanctions. After 21 days had passed and the farmer had not withdrawn or corrected the complaint, the neighbor sought to dismiss the farmer's complaint for failure to state a claim and sought sanctions against the farmer in the same motion. How is the court likely to rule on the motion for sanctions? A. deny it, bc represented parties are not subject to sanctions under Rule 11 B. deny it, bc the neighbor did not seek sanctions in a separately filed motion C. grant it, bc the neighbor first served the motion on the farmer more than 21 days prior to filing the motion with the court D. grant it, bc farmer's attorney failed to conduct a reasonable refiling inquiry and asserted claims that lacked sufficient basis in the law

B

A man from State A initiated a products liability action against ABC, Inc., a State B corporation with its principal place of business in State B for injuries suffered in an accident involving a car manufactured by ABC. The man's theory of liability was that the airbag failed to deploy properly. The man seeks $100,000. As it turns out, XYZ, Inc.—a State A corporation headquartered in State B—supplied the airbags used in the ABC cars. ABC felt that XYZ should have been joined in this action as a co-defendant. ABC thus filed a motion to dismiss the action for failure to join XYZ as a required defendant in this case. How should the court rule on this motion? A. Deny it, because although joinder of XYZ is not feasible, the court can shape relief to lessen any prejudice. B. Deny it, because it is not necessary for all joint tortfeasors to be named as defendants in a single action. C. Grant it, because in XYZ's absence the court cannot accord complete relief to the man. D. Grant it, because joinder of XYZ is not feasible given the lack of complete diversity between the man and XYZ.

B

A mother initiated an action in State A federal court against an informant in federal court for wrongful death in connection with the death of the mother's son. In the complaint, the mother stated her claim as follows: "The informant submitted a statement to members of a local gang falsely implicating my son in a recent theft of some of the informant's property and asked the gang to retaliate against my son by killing him. After this conversation with the informant, the gang members located my son and beat him to death as punishment for the theft they were told he committed. Based on these actions, the informant is liable for wrongful death under State A law." Under State A's wrongful death law: (1) surviving parents are permitted to bring wrongful death actions; (2) wrongful death actions are permitted only against persons directly responsible for causing the death; and (3) a recent decision by the Supreme Court of State A held that persons who "expressly solicit" others to kill the decedent may be held liable under the wrongful death law. If the informant responds to the complaint by filing a motion to dismiss this action under Rule 12(b)(6) is the court likely to grant the motion? A. no, bc the pleadings have not closed yet B. no, bc the facts alleged in the complaint claim that the informant asked the gang to kill the son C. yes, bc the facts alleged in the complaint do not show that the informant is directly responsible for the son's killing D. yes, bc the mother's allegations do not show plausible entitlement to relief

B

A partner in a law firm left the firm to start a new law firm, taking several clients along. The old law firm initiated an action in State A state court against the departing partner for breaching the partnership agreement. The partner responded by removing the action to State A federal court. The law firm moved for remand, which the federal court denied on June 1, 2016. By September 1, 2016, the departing partner had not filed any further pleadings or motions in the case. The law firm thus sought and obtained an entry of default from the clerk on September 5, 2016. The law firm then applied for default judgment before the court; the court set a hearing date of September 15, 2016. The law firm served notice of the hearing on the departing partner on September 10, 2016. On September 15, 2015 the departing partner appeared at the hearing and opposed the entry of a default judgment. How is the court likely to rule on the application for default judgment? A. deny it, bc a default judgment cannot be entered once a party has appeared by removing a case to federal court B. deny it, bc although the departing partner failed to put forward a defense in the action, the departing partner did not receive sufficient notice of the hearing on the default judgment motion C. grant it, bc the departing partner failed to file an answer or motion raising a defense in response to the complaint w/in the required timeframe D. grant it, bc the departing partner received sufficient notice to afford the opportunity to be heard at the hearing on the application for default judgment

B

A pedestrian from State A brought a federal diversity action against a driver from State B for negligence. The complaint alleged that the driver failed to stop at a stop sign and then hit the pedestrian; the pedestrian seeks $100,000 in damages. A passenger from State A who was riding in the car with the driver was also injured in the same accident and wishes to intervene to assert a claim against the driver for $50,000. Should the court permit the passenger to intervene in this action? A. No, because the permitting intervention is likely to cause juror confusion. B. No, because the there would be no jurisdiction over the passenger's claim. C. Yes, because the interests of the passenger would be impaired if the action proceeded in the passenger's absence. D. Yes, because the claims of the passenger and the pedestrian share common questions of law and fact.

B

A pilot filed a federal diversity action against an airplane manufacturer asserting a products liability claim. The basis for the pilot's claim was that the manufacturer had failed to install a particular type of landing gear. In fact, the airplane in question was actually equipped with the landing gear that the pilot alleged was not installed. Upon reviewing the complaint, the manufacturer filed a motion for sanctions with the court. How should the court rule on the motion? A. Deny it, because sanctions are inappropriate for mere factual disputes. B. Deny it, because the pilot was not previously served the motion more than 21 days prior to it being filed. C. Grant it, because the pilot failed to conduct a reasonable prefiling inquiry. D. Grant it, because the pilot's complaint contained a factual contention that lacks evidentiary support.

B

A plumber filed a breach of contract claim against a homeowner in federal court. The homeowner responded by filing an answer in which the defenses of lack of subject matter jurisdiction and insufficient service of process were raised. Has the homeowner waived this latter defense? A. no, bc a motion challenging service of process may be made at any time B. no, bc the homeowner has not previously filed any motions under Rule 12 C. yes, bc by responding with an answer, the homeowner revealed that he had received actual notice of the action D. yes, bc challenges to service of process must be made in a pre-answer motion or they are waived

B

A warden from State A was allegedly defamed by an article written by a reporter from State A and published in the Daily News, which is owned by Daily News Inc., a State B corporation headquartered in State B. The warden initiated an action in State A federal court against Daily News, Inc., seeking $100,000 in damages. Daily News filed a motion to dismiss the warden's claim for failure to join the reporter as a defendant in the case. How should the court rule on this motion? A. Deny the motion, because nonjoinder is not a ground for dismissing an action. B. Deny the motion, because the reporter's presence in the action is not required to protect the reporter's interests or the interest of the parties. C. Grant the motion, because the case can proceed against the Daily News and the reporter in one action in state court. D. Grant the motion, because the reporter and the warden are both from State A, making the joinder of the reporter not feasible.

B

ABC, Inc. and XYZ Corp. were in negotiations to merge into a single company. Unfortunately, terms could not be reached and the companies could not consummate the deal. Subsequently, ABC began selling a product that appeared to incorporate trade secrets of XYZ. XYZ initiated an action in State A federal court against ABC for trademark infringement, theft of trade secrets, and breach of contract. ABC responded with a motion challenging personal jurisdiction. XYZ filed a notice of dismissal and refiled the action in State B federal court. ABC responded with an answer in which it raised the expiration of the statute of limitations period as a defense. XYZ moved for an order dismissing its complaint, which the court granted. XYZ then refiled the action in State C federal court where a longer statute of limitations period would apply. ABC responded with a motion for summary judgment asserting the dismissal of the action in State B federal court as res judicata, barring the relitigation of the claim in State C federal court. State B does not treat voluntary dismissals as preclusive of future actions based on the same claims. Is the current State C action precluded by the previous dismissal in State B federal court? A. no, bc State B does not treat such dismissals as claim preclusive B. no, bc the State B federal court's dismissal was not rendered based on the substantive merits of XYZ's claims C. yes, bc under the "two-dismissal rule" the second dismissal-- which occurred in State B federal ct--operates as an adjudication on the merits D. yes, bc court-ordered dismissals other than for lack of jurisdiction, improper venue, or failure to join a necessary party operate as an adjudication on the merits

B

An automaker produced a car called the T2. The T2 was nothing but a replica of the earlier and discontinued T1 with completely redesigned headlamps and taillights. A customer who owned a T2 was in an accident in which the gas tank of the T2 exploded. The customer initiated an action in federal court against the automaker asserting a products liability claim and seeking $100,000 in damages. During discovery, the customer requested "All correspondence regarding the design and placement of the gas tank in the T1 model." If the automaker were to move for a protective order against this request (after meeting and conferring with the customer), how would the court be likely to rule? A. Deny the motion, because all such documents would relate to the subject matter of this action. B. Deny the motion, because such documents would be relevant to the customer's claim in the action. C. Grant the motion, because this action is about the T2, not the T1. D. Grant the motion, because such records would constitute confidential trade secrets.

B

On January 15 an investor filed and served a securities fraud action in federal court against a broker, alleging that the broker misrepresented material facts pertaining to an investment offer made as a private placement via a confidential offering memorandum. Assume that under federal law, securities fraud claims are only available with respect to registered public offerings, not private placements. On February 1 of that same year the broker responded to the complaint by filing a motion for sanctions under Rule 11 with the court. Should the court grant the motion? A. no, bc the proper response would be for the broker to file a motion to dismiss for failure to state a claim B. no, bc the broker failed to serve the motion on the investor more than 21 days prior to filing it with the court C. yes, bc the investor asserted a claim that was not warranted by existing law or by a non frivolous argument for extending, modifying, or reversing existing law or for establishing new law D. yes, bc sanctions are needed to deter repetition of the conduct or comparable conduct by others similarly situated

B

The driver of a car brought an action against a pedestrian for negligence in State A federal court by filing a complaint on June 1, 2017. On June 23, 2017 the driver served a copy of the summons and complaint by personally delivering them to the pedestrian's home in State A. Because the pedestrian was not there, the process server gave the summons and complaint to the 17-year-old babysitter who answered the door. The babysitter lives down the street from the pedestrian. After receiving the complaint from the babysitter, the pedestrian filed a motion to dismiss for lack of personal jurisdiction. The court denied the motion. The pedestrian then filed an answer denying all of the allegations in the complaint and raising the defense of insufficient service of process. How should the court respond to the defense? A. reject the defense, bc a 17-yr-old person found at the home of the defendant is a person of suitable age and discretion B. reject the defense, bc the pedestrian waived his right to challenge the sufficiency of service of process by previously filing a motion to dismiss for lack of personal jurisdiction C. dismiss the action, bc delivery to a defendant's dwelling must be made on someone who is an adult, not a minor as was the case here D. dismiss the action, bc delivery to a defendant's dwelling must be made on someone who resides there, which was not done here

B

The plaintiff, a marshmallow sales representative from State A, sued the defendant, a citizen of State B, in federal court for extensive injuries that the plaintiff received from a fight with the defendant in an elevator. The plaintiff's medical bills totaled $15,000, and he also alleges $70,000 in damages for pain and suffering. The defendant has a claim against the plaintiff for breach of contract, in that one of the plaintiff's deliveries turned out to be melted marshmallows. The defendant alleges $76,000 in damages. May the defendant have his claim heard as a counterclaim in the plaintiff's action? A. Yes, because the court may exercise supplemental jurisdiction over the claim. B. Yes, because the defendant may bring the claim as a permissive counterclaim. C. No, because the court cannot exercise supplemental jurisdiction over the claim. D. No, because the court does not have subject matter jurisdiction over the underlying claim.

B

A passenger initiated an action in federal court against a bus company seeking damages for injuries sustained when a bus operated by the company crashed into a ravine. The driver of the bus was critically injured but survived the accident initially. Attorneys for the passenger visited the bus driver in the hospital to interview him about the accident. The attorneys took extensive notes documenting the driver's explanation of what happened leading up to the accident. Sadly, the bus driver died from his injuries two days later. During discovery in this case, the bus company issued a request for the notes of the bus driver interview prepared by the passenger's attorneys. After meeting and conferring about the request, the passenger filed a motion for a protective order that would permit the withholding of the notes. How should the court rule? A. Deny the motion, because the attorney-client privilege has been waived. B. Deny the motion, because the bus company has substantial need for the notes (allowing that any mental impressions will be protected from disclosure). C. Grant the motion, because the notes are protected by the attorney-client privilege. D. Grant the motion, because the notes are protected by the work-product doctrine.

B The notes constitute attorney work product prepared in anticipation of this litigation. However, they are nonetheless discoverable because the driver's death makes him no longer available as a witness, giving the bus company a substantial need for the notes. Fed. R. Civ. P. 26(b)(3)(A)(ii). That said, the passenger's attorney will be able to protect from disclosure any mental impressions, conclusions, opinions, or legal theories contained within the notes. Fed. R. Civ. P. 26(b)(3)(B).

ABC Inc. initiated an action against XYZ Corp. for patent infringement. XYZ retained an expert to advise its lawyers on the scientific and technical aspects of the case. In the course of this relationship, the XYZ attorneys shared with the expert copies of design plans for the allegedly infringing device and asked the expert to determine whether the device relied upon or incorporated ABC's patented process. The expert prepared a report for the XYZ attorneys outlining her views on that topic. However, because this expert would not testify as a witness at trial, XYZ did not disclose the report to ABC. During discovery, ABC requested and the court issued a subpoena to XYZ's expert to appear at a deposition and to bring "any document in which the expert reported his or her evaluation of the allegedly infringing device and patented process at issue in this case and any material shared with you by XYZ that you considered in forming your opinion." Which of the following represents the strongest ground for objecting to this subpoena? A. Experts may not be deposed until after the required written expert report is disclosed. B. Non-testifying experts are immune from being deposed absent exceptional circumstances. C. The requested documents are protected from disclosure by the attorney-client privilege. D. The requested documents are protected work product.

B Non-testifying experts may not be deposed unless it is to discuss a physical or mental examination conducted pursuant to Rule 35(b), or there are exceptional circumstances justifying such a deposition, meaning that the information the expert possesses is otherwise unobtainable. Fed. R. Civ. P. 26(b)(4)(D). Such circumstances do not exist in this case.

You represent a major retailer involved in litigation against a supplier for breach of contract. The retailer would like to obtain some answers to two straightforward questions from a competitor to determine whether the supplier provided supplies to the competitor that had been promised to the retailer. Those questions would be "In the past year, did your company receive X supplies from the supplier?" and "Were you aware that the supplier had previously agreed to provide X supplies to the retailer?" The competitor is acrimonious toward your client and is not likely to be willing to answer these questions. Which of the following would be the best way to obtain answers to these questions from the competitor? A. Request that the competitor prepare an affidavit addressing the two questions. B. Serve on the competitor a subpoena to appear at a deposition at which the two questions will be posed. C. Serve on the competitor interrogatories posing the two questions. D. Serve on the competitor requests to admit or deny the allegations underlying the two questions.

B Rules 30 and 31 permit the deposition of any person or entity, which may include nonparties pursuant to Rule 45. The competitor will have to answer the questions under oath

A 13 year-old student was standing on the running board of a truck during a parade when the handle she was holding detached from the vehicle, causing her to fall and hit her head on the ground. She initiated an action against the truck manufacturer in federal court seeking $100,000 in damages. The manufacturer responded with an answer denying any liability for the detached handle and asserting that the student assumed the of risk of harm associated with her conduct by standing on the exterior running board while the truck was in motion. Under the applicable state law, a defendant asserting an assumption of risk defense must establish that the plaintiff (1) had actual knowledge of the danger, (2) understood and appreciated the risks associated with such danger, and (3) voluntarily exposed herself to those risks. Further, assumption of risk is not an available defense under this state's law for minors aged 14 and under. You are the attorney for the student. How should you respond to the defendant's answer? A. seek permission to file a reply in which you deny the assumption of risk allegation B. file a motion for judgment on the pleadings based on the legal insufficiency of the assumption of risk allegation C. file a motion to strike the assumption of risk allegation from the answer D. file a motion to dismiss the manufacturer's answer for failure to state a claim

C

A corporation has been the subject of several news reports charging that the pollution put out by its factory endangers 5,000 residents of a subdivision. The corporation filed an action naming all 5,000 residents as defendants and seeking a declaratory judgment that it is not liable for damaging their health, and the court properly certified the action as a class action. Per court order, detailed notice of the suit is posted on every lightpole throughout the subdivision, but the residents are not notified individually. Was notice in this case constitutional? A. Yes, because the posting was reasonably calculated to give notice to all 5,000 residents. B. Yes, if a state statute authorizes such process. C. No, because all residents could reasonably be notified by mail. D. No, because individual defendants must always be notified by personal service of a summons.

C

A customer who had fallen in front of a store initiated an action in federal court against the owner of the store for negligently maintaining the walkway in front of the store to prevent icy and dangerous conditions from harming its customers. The storeowner filed an answer alleging in paragraph 10 that the customer was contributorily negligent because the customer was spinning on the walkway with her eyes closed at the time she fell (assume such facts constitute contributory negligence under the applicable law). The court ordered the customer to file a reply to respond to the contributory negligence defense. The customer filed a reply that stated, "The allegations in paragraph 10 are legal conclusions to which no response is due." Under the applicable law, contributory negligence would serve as a complete bar to recovery by the customer. You represent the storeowner in this case. What is the best strategy for handling this case on behalf of the storeowner at this point to prevail? A. file a motion for an entry of default and default judgment B. file a motion for an involuntary dismissal against the plaintiff C. file a motion for judgment on the pleadings D. file a motion to dismiss the complaint for failure to state a claim

C

A mayor sued a newspaper for defamation in federal court. The mayor filed and served the complaint five days before expiration of the applicable two-year statute of limitations period. The newspaper responded with an answer to the complaint in which it denied all of the mayor's material allegations. Twenty days after serving the answer on the mayor, the mayor filed an amended complaint adding the reporter who wrote the article containing the allegedly defamatory statement as a defendant in the action. The reporter responded by filing a motion to dismiss the mayor's claim for failure to state a claim based on expiration of the applicable statute of limitations period. Which of the following would be an appropriate ruling of the court on this motion? A. Deny it, because the amendment adding the reporter relates back to the time of filing, which preceded the expiration of the applicable statute of limitations period. B. Deny it, because the mayor is permitted to amend the complaint in this manner as a matter of course. C. Grant it, because the claim was asserted after the expiration of the applicable statute of limitations period. D. Grant it, because the time to amend the complaint as a matter of course has expired.

C

A motorcyclist initiated an action in federal court against a motorcycle manufacturer for harm suffered in an accident allegedly caused when a motorcycle made by the manufacturer exploded after a collision. After being served the complaint, the manufacturer impleaded the company that supplied the gas tank for the motorcycle, seeking indemnification for any liability the motorcycle manufacturer might have to the motorcyclist (the supply contract between the manufacturer and the gas tank company contained an indemnification agreement). The motorcycle manufacturer also asserted a claim for breach of contract against the gas tank company based on the alleged supply of substandard gas tanks. Over the course of the next month, the case was litigated and the motorcycle manufacturer was found not liable. The manufacturer also prevailed on its breach of contract claim against the gas tank company. One month later, the gas tank company asserted a claim in federal court against the motorcycle manufacturer for money owed under the supply contract; the manufacturer had not paid for supplied gas tanks for several years. The manufacturer sought to dismiss this action as barred for not having been raised in the previous action. How is the court likely to rule on this motion? A. deny it, bc the gas tank company claim does not arise out of the same transaction or occurrence as the motorcyclist's claim against the motorcycle manufacturer in the previous action B. deny it, bc the gas tank companys claim does not arise out of same transaction or occurrence as the indemnification claim previously asserted by the motorcycle manufacturer C. grant it, bc the gas tank company claim arises out of the same transaction as the breach of contract claim asserted by the motorcycle manufacturer in the previous action D. grant it, bc the gas tank companys claim shares a common question of law with the breach of contract claim asserted by the motorcycle manufacturer in the previous action

C

A nuclear power plant experienced severe damage after an earthquake, resulting in the emission of radioactive material over an eighty-mile radius. Unfortunately, this occurred near a major metropolitan area, meaning that millions of residents were affected by the emitted radiation. An attorney experienced in litigating claims in the wake of these types of accidents began recruiting affected citizens to form a class that would seek compensation for the harm they suffered as a result of this incident. After conducting numerous medical examinations of prospective clients, the attorney concluded that each had medical expenses ranging from $10,000 to $100,000. The attorney initiated a class action on behalf of a plaintiff from State A who suffered $100,000 in medical expenses and all similarly situated people who were exposed to the radiation after the earthquake. The total amount of damages sought was $2.5 billion. Under a federal statute, however, the liability of the company that owned the nuclear power plant is limited to $500 million. The company that owns the power plant opposes certification of the class. Which of the following would be an appropriate ruling on the class certification issue? A. Deny class certification, because the named plaintiff's claim is not typical of those absent plaintiff's whose claims are worth only $10,000. B. Deny class certification, because the individual issues are likely to predominate over the common issues. C. Grant class certification, because individual suits would be likely to adversely impact the ability of other litigants to recover damages from the company. D. Grant class certification, because individual suits would be likely to establish incompatible standards of conduct for the company with respect to individual litigants.

C

A patient from State A initiated an action in federal court against a State B doctor and a State C hospital for injuries sustained during a recent operation. The hospital in turn asserted an indemnification claim against the doctor, claiming that its contract with the doctor required the doctor to reimburse the hospital for any liability it incurred resulting from the doctor's wrongdoing. The doctor responded by asserting a claim against the hospital for an unrelated debt of $100,000. May the doctor's debt claim be joined in this action? A. no, bc it does not arise from the same transaction or occurrence as any of the other claims asserted in the action B. no, bc it does not seek to hold the hospital responsible for all or part of the liability asserted against the doctor C. yes, bc the doctor may respond to the hospitals claim with any claims that she has, related or unrelated D. yes, bc the hospital has consented to jurisdiction by asserting a claim against the doctor

C

A pedestrian initiated an action against a car driver in federal court seeking damages for negligence that resulted in a collision between the two. The car driver thereupon filed a third-party complaint against a truck driver seeking indemnification for the potential liability that the car driver faced in this action based on the car driver's claim that the truck driver was responsible for causing the collision (assume that under the applicable law, joint tortfeasors may be subject to indemnification claims). After the truck driver was joined in the action, the truck driver asserted a claim against the pedestrian for negligence and sought damages for injuries suffered in the accident. Is the truck driver's claim against the pedestrian properly joined in this action? A. no, bc the truck driver has not obtained the permission of the ct B. no, bc the pedestrian is not currently an opposing party, having not yet asserted any claim against the truck driver C. yes, bc the truck drivers claim against the pedestrian arises out of same occurrence as the pedestrian's claim against the car driver D. yes, bc the truck drivers claim against the pedestrian shares a common question of fact with the pedestrian's claim against the car driver

C

A pilot filed a breach of contract claim against an airline in federal court. The airline responded with an answer in which it raised the defenses of a lack of personal jurisdiction and improper venue. The court rejected both defenses. After the case was submitted to the jury, the airline sought to dismiss the action for lack of subject matter jurisdiction. May the court consider this defense at this point in the proceedings? A. no, bc airline previously filed motions to dismiss for lack of personal jurisdiction and improper venue B. no, bc a motion to dismiss for lack of subject matter jurisdiction must be made prior to the conclusion of the trial C. yes, bc a motion to dismiss for lack of subject matter jurisdiction may be made at any time D. yes, bc the jury has not yet rendered its verdict

C

A plaintiff is suing a defendant in federal court for personal injuries arising out of an assault and battery. The defendant has confided in his therapist certain matters that the plaintiff believes would be relevant to her case and helpful in finding further information. On request by the plaintiff, will the federal court order discovery of what the defendant told his therapist over the defendant's objection? A. Yes, because the information from the therapist will lead to other discoverable evidence. B. Yes, if the judge orders that the records from the therapist be sealed. C. No, because the communication with the therapist is privileged. D. No, unless the therapist waives the privilege.

C

A worker was injured while carrying a set of boxes on a pier when a forklift hit him. Based on markings located on the forklift stating "Property of ABC," the worker initiated an action in federal court against ABC, Inc. on March 1. In the complaint, the worker alleged in paragraph 5 as follows: "On [date] a forklift owned and operated by ABC, Inc. collided with the plaintiff, resulting in serious injuries." Two weeks later on March 15 ABC responded to paragraph 5 in its answer by stating, "ABC denies the allegations in paragraph 5." That same day, ABC sent a copy of the complaint to XYZ Corp., a company that had been leasing the forklift in question at the time of the accident. Three months later during a deposition, the President of ABC remarked that it was not operating the forklift in question at the time of the accident but rather was leasing the forklift to XYZ Corp. Upon learning this, on June 15 of the same year the worker sought to amend its complaint to substitute XYZ as a defendant. Unfortunately, the statute of limitations period had expired on the previous May 1 and XYZ objected to the amendment on that ground. The worker responded by arguing that the amendment should relate back to the time of filing of the complaint. Will the amendment adding XYZ as a party relate back? A. no, bc XYZ was not named as a defendant in the action until after 90 days had passed from the time the complaint in this action was originally filed B. no, bc ABC's denial gave the worker notice that it had named the wrong company as a defendant and thus could have determined the proper defendant within the statute of limitations period C. yes, bc XYZ had notice of the action within 90 days of its service on ABC that but for a mistake concerning the proper identity of the forklift operator XYZ would have been named as a defendant originally D. yes, bc ABC knew that XYZ should have been named as a defendant when it filed its answer

C

ABC, Inc. (a State A corporation) initiated an action in federal court against a State B retailer seeking to set aside a contract between the retailer and XYZ, Inc. (a State A corporation) because ABC believed the contract violated its exclusive supply agreement with the retailer. After filing its answer, the retailer filed a motion to dismiss the claim based on the failure to join XYZ as a necessary party defendant in the action. How should the court respond to the motion? A. Deny the motion, because the dispute between the retailer and ABC can be resolved completely without the presence of XYZ. B. Deny the motion, because the retailer has waived the ability to raise this motion at this point in the proceedings. C. Grant the motion, because XYZ's rights under its contract with the retailer will be prejudiced if the court grants the relief ABC seeks, XYZ's joinder is not feasible because it would destroy complete diversity, and the court cannot otherwise protect XYZ's interests. D. Order the joinder of XYZ as a party defendant, because XYZ's rights under its contract with the retailer will be prejudiced if the court grants the relief ABC seeks.

C

ABC, Inc. constructed a building for XYZ, Inc. in State A. The building was completed over one year after the time the two parties had agreed for its completion. In light of this delay, XYZ initiated an action in federal court against ABC seeking liquidated damages under their contract. ABC's liability insurer, Ensure, Inc., had an agreement with ABC to indemnify it for any liability that might arise out of its construction projects. Thus, twelve days after being served with XYZ's complaint, ABC filed a third-party complaint against Ensure, claiming that it would be liable for the entire amount claimed by XYZ were ABC found liable in the action. ABC also filed a separate third-party complaint against SubCon, Inc., the subcontractor that worked on the ABC building project for XYZ, as a defendant in this third-party complaint, asserting that its negligence was responsible for the construction delay and thus it is liable for any damages that might be assessed against ABC (assume that in State A, joint tortfeasors may be subject to indemnification/contribution claims). SubCon objects to being joined in the action in this way. Is joinder of SubCon permissible under the Federal Rules? A. no, bc the claim against it does not share a common question of law or fact w/claim against Ensure B. no, bc ABC did not seek permission of the ct to file the third party complaint C. yes, bc ABC is claiming that SubCon is liable to ABC for the potential liability it facts on the claim by XYZ D. yes, bc the claim against SubCon arises out of same transaction and occurrence that is the subject matter of the claim ABC is asserting against Ensure E. yes, bc the claim, against SubCon arises out of the same transaction and occurrence that is the subject matter of the claim XYZ is asserting against ABC

C

After an employee retired, the employer was obligated to transfer the funds held in a company-owned retirement fund into an account designated by the employee in a lump sum payment of $325,560.24. When the deposit was never made and after numerous letters and inquiries, the employee initiated an action in federal court against the employer for breach of contract, seeking $325,560.24. Service of the summons and complaint on the employer occurred on September 1, 2017. The employer responded with a motion to dismiss for lack of subject matter jurisdiction, which the court denied on September 30, 2017. By December 1, 2017 there had been no further response from the employer. Thus, the employee sought entry of default and default judgment against the employer, attaching an affidavit showing that $325,560.24 was due from the employee's retirement account with the employer. The clerk entered default and default judgment for that amount on December 15, 2017. The next day, on December 16, the employer moved the court to set aside the entry of default judgment. How is the court likely to rule? A. deny the motion, bc the employee proved that the employer was liable for payment of the funds, making the employee entitled to a judgment to that effect B. deny the motion, bc the employer failed to plead as required and the claim was for a sum certain C. grant the motion, bc although a clerk can enter a default, the court-- not the clerk-- must enter a default judgment under these circumstances D. grant the motion, bc the employer's appearance to make a motion to dismiss precludes the entry of default

C

An entrepreneur wanted to retain a certified public accountant (CPA) to assist him with preparation of tax return forms. Although not licensed to practice law, the CPA was a graduate of the local law school, where she had focused on all aspects of state and federal tax law. During their initial meeting, the entrepreneur asked if he had to declare income received for serving as the member of the board of directors of a company based in Bermuda. The CPA responded that he would have to report it on his state and federal tax forms. The entrepreneur decided not to retain the CPA and instead prepared the tax returns on his own. The entrepreneur did not report the Bermuda board income on either the state or federal tax returns. The Commissioner of the Internal Revenue Service initiated a tax deficiency action against the entrepreneur for taxes owed on the unreported Bermuda board income. During discovery, the Commissioner called for a deposition of the CPA. The Commissioner's attorney asked the CPA to discuss her conversation with the entrepreneur regarding the Bermuda board income. Does the attorney-client privilege protect the CPA from answering this question? A. Yes, because the entrepreneur sought legal advice from the CPA. B. Yes, because CPA is a law school graduate and the communication was confidential. C. No, because the CPA was not a licensed attorney during the conversation in question. D. No, because the entrepreneur sought tax advice—not legal advice—from the CPA.

C

An inventor from State A got notice that a newspaper was about to publish the secret components of an invention. This publication was going to be in conjunction with a television station that planned to air a story revealing the same information. The newspaper is owned by Daily News Inc., a State B corporation headquartered in State B. The television station is owned by ABC, Inc., a State A corporation headquartered in State B. The inventor initiated an action in State A federal court against Daily News, Inc., seeking an injunction preventing it from publishing the article. Daily News filed a motion to dismiss the inventor's claim for failure to join ABC as a defendant in the case. Which of the following would be an appropriate ruling of the court on this motion? A. Deny the motion, because ABC's presence in the action is not required to protect the inventor's interests or the interest of the parties. B. Deny the motion, because nonjoinder is not a ground for dismissing an action. C. Grant the motion, because ABC's presence is required to accord complete relief to the inventor but their common State A citizenship makes joinder not feasible. D. Grant the motion, because proceeding in ABC's absence is likely to result in Daily News being subject to inconsistent obligations.

C

The driver of a car and a passenger in the car initiated a products liability action in federal court against the manufacturer of the car for injuries they each sustained when the car exploded after a mild collision. The passenger subsequently asserted a negligence claim against the driver based on the driver's actions during the same collision. Is the passenger's negligence claim properly joined? A. no, bc there is no common question of law or fact uniting the passengers claim and claims against the manufacturer B. no, bc the passenger assumed risk of injury by consenting to be a passenger in the drivers car C. yes, bc the passengers claims arises out of same occurrence that is subject matter of claims asserted against the manufacturer D. yes bc the passenger is free to join as many independent claims as it has to claim it is asserting against manufacturer

C

The owner of a nursery sued a manufacturer in federal court for damage to his growing trees after one of the manufacturer's factories negligently emitted toxic gas. In interrogatories answered on May 1, the nursery owner truthfully stated that the leaves on his red maple trees are normally out by April 15, but this year the red maples do not have any leaves. A week later, on May 8, all of the nursery owner's red maples have normal leaves on them. Is he under a duty to supplement the responses he gave in the interrogatory? A. No, unless the manufacturer asks for an update during the discovery period. B. No, because the information provided in the interrogatory was true. C. Yes, because the nursery owner knows the information provided in the interrogatory is no longer true. D. Yes, unless the manufacturer could find the information on its own.

C

A State A developer who claimed to possess an easement giving it access to land where a new building was to be constructed initiated a declaratory judgment action in federal court against the State B owner of the land across which the easement traversed. The developer sought a declaration of the validity of its easement and an injunction ordering the landowner to remove a gate that was obstructing the developer's use of the easement. The co-owner of the land (a State A citizen) was not joined as a party in this action. The State B owner filed a motion to dismiss for failure to join the State A co-owner as a party defendant in this action. How should the court rule on this motion? A. Deny the motion, because the absent co-owner will not be bound by the outcome and will be free to pursue its interests in a subsequent action. B. Deny the motion, because although the ability of the absent co-owner to protect its interest in the land will be impaired by a court order concerning the land, the court can simply order the joinder of the co-owner as a party defendant given that there would be supplemental jurisdiction over the developer's claim against the co-owner. C. Grant the motion, because a judgment rendered in the co-owner's absence will subject the developer to a substantial risk of incurring inconsistent obligations and the co-owner's State A citizenship makes joinder not feasible. D. Grant the motion, because the ability of the absent co-owner to protect its interest in the land will be impaired by a court order concerning the land and the co-owner's State A citizenship makes joinder not feasible.

D

A State A patron of a department store located in State B was injured when the escalator malfunctioned while the patron was riding it. The patron initiated a negligence action in federal court in State A seeking $100,000 in damages for personal injuries. After receiving service, the department store timely moved to dismiss the action for lack of personal jurisdiction on April 1, 2016. On April 5, 2016 the patron filed a notice of dismissal and refiled the complaint in the federal court in State B. Were the patron's actions permitted under the Federal Rules? A. no, bc once the department store responded with its motion to dismiss, the patron needed the permission of the court or consent of the department store to voluntarily dismiss the case B. no, bc the State A federal ct dismissal is with prejudice, precluding refiling in State B federal ct C. yes, bc the patron is the master of the complaint and can choose to dismiss it at any time D. yes, bc the department store had not yet pleaded or sought judgment in the action

D

A car owner from State A initiated an action against a State B auto manufacturer in federal court seeking $1 million for damages for injuries suffered when the owner's car exploded in a collision. The car owner asserts that the manufacturer was negligent in its design and placement of the gas tank. After reading about the lawsuit in the newspaper, another owner of the same kind of car from State C, sought to intervene to assert a claim against the manufacturer for damages arising out of an explosion involving the car during a separate accident. The State C owner's claim alleges negligent design and placement of the gas tank and seeks $80,000. May the State C owner's motion to intervene be granted? A. No, because the State C owner's claim arose out of a separate accident. B. No, because the State C owner's claim does not share a common question of law or fact with the State A owner's claim against the manufacturer. C. Yes, because disposition of the State A owner's claim in the absence of the State C owner may as a practical matter impair or impede the State C owner's ability to protect its interests. D. Yes, because the State C owner's claim shares a common question of law or fact with the State A owner's claim against the manufacturer.

D

A customer developed a skin rash after using lotion made by ABC, Inc. The customer initiated a products liability action in federal court seeking $100,000. ABC was served the summons and complaint on June 1, 2017. On June 20, 2017, ABC filed a motion to dismiss for lack of personal jurisdiction, which the court denied on July 1, 2017 (the parties learned of this ruling that same day). On August 1, 2017, after failing to receive an answer to the complaint from ABC, the customer sought entry of default by the clerk of the court, which the clerk entered. Was entry of default appropriate under these circumstances? A. no, bc ABC did not receive notice prior to entry of default B. no, bc ABC made an appearance in this case C. no, bc default must be entered by a judge, not the clerk D. yes, bc ABC failed to file an answer within the required timeframe E. yes, bc ABC's motion to dismiss was not filed within required timeframe

D

A homeowner from State A initiated a breach of contract action in federal court against a contracting company organized as an LLC headquartered in and organized under the laws of State A. The members of the LLC are a State B plumber and a State C carpenter. The homeowner seeks $80,000 on the claim. Formal service of process was achieved on June 1 under the provisions of Rule 4. By August 1 of the same year, the contracting company had not filed an answer. On August 2, the homeowner sought an entry of default, which the clerk entered. On August 3, the homeowner applied to the court for a default judgment. In response, on August 5 the court held a hearing and then entered default judgment. Were the court's actions appropriate? A. No, because the contracting company was not given notice of the application for default judgment. B. No, because the judgment is void based on a lack of subject matter jurisdiction. C. Yes, because the homeowner's claim is for a sum certain. D. Yes, because the time for the contracting company to file a response has expired and it has not otherwise appeared in this action.

D

A homeowner initiated an action in federal court against a contractor, alleging that the contractor had negligently completed construction of the home the contractor had built for the homeowner. Two months after the pleadings had closed and one week into discovery, the homeowner learned that the wood the contractor used for flooring in the home was not of the quality and origin that the contractor had represented. The homeowner thus sought to file an amended complaint adding a fraud claim against the contractor. The contractor objected to the amendment. How is the court likely to rule on the homeowner's request to amend the complaint? A. deny it, bc more than 21 days have passed since the homeowner was served with the contractor's answer B. deny it, bc the contractor has not consented to the amendment C. grant it, bc the homeowner may amend the complaint once w/o permission of the court D. grant it, bc the homeowner only discovered the fraud in discovery and the contractor will have a fair opportunity to defend itself against the fraud claim

D

A motorcyclist, a taxi driver, and a bus driver got into a three-vehicle accident. The motorcyclist sued the other drivers for personal injuries arising from the accident. In turn, and in the same case, the bus driver asserted a claim against the taxi driver for injuries arising out of that accident, and the taxi driver asserted a claim against the motorcyclist. What are the bus driver's and the taxi driver's claims called? A. The bus driver's claim is a counterclaim, and the taxi driver's claim is a cross-claim. B. Both claims are counterclaims. C. Both claims are cross-claims. D. The bus driver's claim is a cross-claim, and the taxi driver's claim is a counterclaim.

D

A passenger (from State A) in a car initiated an action in federal court against the State B driver of the car based on the driver's negligence in causing a collision and against a State C pedestrian for recklessness based on the pedestrian's jaywalking that contributed to the accident. The driver responded by filing a claim against the pedestrian, asserting that the pedestrian was liable to the driver for recklessness based on the pedestrian's conduct causing to the accident. The pedestrian, in turn asserted a breach of contract claim based on an unrelated contract dispute between the pedestrian and the driver, seeking $100,000. May the pedestrian's breach of contract claim be joined under the Federal Rules? A. no, bc the claim does not arise out of same occurrence that is basis for passengers claim against driver B. no, bc the claim does not assert that the pedestrian is liable to the driver for all or part of the passengers claim against the driver C. yes bc the driver and pedestrian were properly joined as codefendants when the passenger asserted a claim against them based on the same collision D. yes, bc the pedestrian is free to assert any claim against an opposing party

D

A patient filed a breach of contract claim against a doctor in state court. The day after removing the case to federal court, the doctor moved to dismiss the complaint for failure to state a claim and the motion was denied. The doctor now moves to dismiss the complaint for lack of personal jurisdiction. Has the doctor waived this latter defense? A. No, because a motion to dismiss may be made at any time through trial. B. No, because the doctor has yet to file an answer. C. Yes, because the doctor previously filed a notice of removal in this case. D. Yes, because the motion challenging personal jurisdiction must be consolidated with other motions to dismiss and raised initially or it is waived.

D

A pedestrian from State A brought a negligence action in federal court against a State B motorcyclist and a State C truck driver, seeking $100,000 from each for injuries arising out of an accident. The motorcyclist responded by asserting a claim for negligence against the truck driver seeking $50,000 for the motorcyclist's injuries. The motorcyclist also asserted an unrelated claim against the truck driver seeking $30,000 for an alleged breach of contract. As it turns out, the motorcyclist formerly worked as a courier for the truck driver but that arrangement was abruptly terminated several months ago; a dispute over the discontinuation of that contract is the basis for the motorcyclist's breach of contract claim. Is the breach of contract claim properly joined? A. no, bc the breach of contract claim does not arise out of same occurrence that is subject matter of pedestrians negligence claim B. no, bc amount in controversy is insufficient C. yes, bc joinder would promote efficiency and avoid unnecessary cost and delay D. yes, bc the motorcyclist is free to join independent claims to the negligence claim it is asserting against the truck driver

D

A shopper sued a grocery store in federal court for a back injury after a fall in the store. The shopper provided the store with a medical report from his own doctor. However, the store located a physician specializing in back injuries who agreed to examine the shopper independently. The store sent a discovery notice to the shopper instructing him to call the physician to set up an appointment. The shopper declined, asserting that he would not do so without a court order. The store responded that if the shopper insisted on a court order, it will also ask the court to impose sanctions on the shopper for failing to cooperate in discovery. Is the store's position as to sanctions justifiable? A. Yes, because the shopper must cooperate with the store during the discovery period. B. Yes, if the physician is deemed to be a neutral physician. C. No, if the shopper's original doctor was also a back specialist. D. No, because the shopper need not submit to an examination by a physician without a court order.

D

A welder filed a wrongful termination claim against his employer in federal court. The employer moved to dismiss for lack of subject matter jurisdiction, which the court denied. The employer now files an answer in which it raises the defenses of a lack of personal jurisdiction and failure to state a claim. Has the employer waived either of these two defenses? A. both have been waived, bc personal jurisdiction and failure to state a claim challenges must be raised initially B. neither has been waived, bc subject matter jurisdiction challenges can be raised at any time C. only the failure to state a claim challenge has been waived bc it was raised after the court ruled on a previous motion D. only the personal jurisdiction challenge has been waived, bc such challenges must be raised initially

D

ABC, Inc. initiated a trademark infringement action against XYZ Corp., a French company, in federal court. XYZ was served properly in France on August 1, 2016. On September 1, 2016, having not heard any response from XYZ, ABC sought entry of default, which the clerk entered. After learning of the default, on September 5 of that same year XYZ moved to have the default set aside, explaining that its offices had been closed throughout August and September for vacation, which is why it had not yet responded to the complaint. Along with the motion to set aside default XYZ included the answer it proposed to file, which denied the principal allegations and raised a number of meritorious defenses. How should the court rule on the motion to set aside the default? A. deny the motion, bc XYZ was obligated to respond to the complaint within 21 days of service of process B. deny the motion, bc XYZ's reason for failing to respond in a timely fashion does not constitute excusable neglect C. grant the motion, bc XYZ had 60 days to respond to the complaint based on its location outside of the US, a time period that has yet to expire D. grant the motion, bc XYZ's default appears to have been in good faith, it offers to defend against the claim on the merits, and ABC will not be prejudiced in prosecuting its claim

D

After failing to deliver purchased materials in a timely fashion, a wholesaler initiated a breach of contract action in federal court against its supplier. The wholesaler and the supplier were already engaged in a separate breach of contract lawsuit in state court based on the same contract but different events. The supplier responded to the federal complaint with an answer, which it served on the wholesaler on November 1. On November 15 of that same year the state lawsuit reached a final judgment, with a jury finding that the contract at issue was void and unenforceable (assume that the state court had proper jurisdiction and that its judgment is valid in all respects). It is now November 16 of the same year. You represent the supplier in the federal action. Which of the following is the best course of action in light of the recent state court judgment voiding the same contract at issue in the federal case? A. file an amended answer as a matter of course, bc less than 21 days have passed since the original answer was served B. file a counterclaim seeking rescission of the contract based on the state court judgment C. file a motion to dismiss the wholesaler's federal breach of contract claim for failure to state a claim upon which relief can be granted D. file a motion to serve a supplemental answer asserting the state court judgment as preclusive of the issue of the contract's validity in the federal action

D

An attorney in one state represented a plaintiff for personal injuries sustained due to the negligence of the defendant, which occurred on January 20. The statute of limitations for the plaintiff's cause of action is governed by the law of a different state. That statute of limitations is only three months. In other states, the statute of limitations for similar causes of action ranges from two years to five years; in the attorney's home state it is three years. The plaintiff first went to see the attorney on June 15, after the statute of limitations had expired. The attorney, based on his experience in other states, believed that the case was well within the statute of limitations. He drafted a complaint, signed it, and filed it in federal court on June 20. The case was immediately dismissed based on the fact that the statute of limitations had expired. May the attorney be sanctioned? A. No, because he did not know that the plaintiff's case was not warranted by existing law. B. No, because he could have argued that the statute of limitations should be reversed. C. Yes, because the attorney's signature makes him strictly liable for any defects in the complaint. D. Yes, because he should have known that the plaintiff's case was not warranted by existing law.

D

An auto manufacturer initiated an action against a steel supplier over the quality of the steel sold to the manufacturer for its cars. After the pleadings were closed, the judge held a pretrial conference at which she ordered the parties and their attorneys to attend in person to discuss settlement. The auto manufacturer sent its lawyer to the conference while the steel supplier sent its lawyer and CEO. The judge informed the auto manufacturer's lawyer that she had ordered a representative from the auto manufacturer to attend the conference, not just their lawyer. The lawyer responded by indicating that a representative of the auto manufacturer could be reached at any time by telephone to discuss any matter. After providing notice and the opportunity to be heard, the court responded by imposing sanctions against the auto manufacturer, ordering it to pay the expenses of the steel supplier associated with attending the conference as well as any associated attorney's fees. The court also rescheduled the conference and ordered the auto manufacturer to appear. Was the court's order for sanctions appropriate? A. No, because monetary sanctions may not be imposed against a represented party under these circumstances. B. No, because parties are permitted to be either present or reasonably available by other means at a pretrial conference. C. Yes, because participating in pretrial conferences by telephone is prohibited under the rule. D. Yes, because the court was empowered to require the attendance of the auto manufacturer at the pretrial conference and impose such sanctions when the auto manufacturer failed to send a representative of the company to appear in person.

D

An employee initiated an employment discrimination claim against the employer in State A federal court. The employer responded with a motion to dismiss for failure to state a claim, which the court granted. The court dismissed the employee's complaint "without prejudice." The employee refiled the exact same complaint in State B federal court. On its own initiative, the district court ordered the plaintiff to file an amended complaint that differed from the one dismissed in the State A federal court action. The deadline for doing so was set as September 1, 2017. By November 1, 2017 the plaintiff had failed to file an amended complaint as ordered. The court entered an order dismissing the case on that date, "with prejudice." Was the court's dismissal appropriate? A. no, bc the employer has not moved for an involuntary dismissal and courts are not empowered to dismiss cases sua sponte B. no, bc the dismissal of the complaint in State A was without prejudice C. yes, bc courts are empowered to eliminate frivolous claims on their own initiative D. yes, bc the plaintiff failed to comply with the court's order to replead

D

An individual investor lost $80,000 when it was revealed that the fund the investor had invested in was a fraudulent scheme. This amount constituted the life savings of this investor, leaving the investor with limited financial resources. The investor initiated an action against the bank that ran the fund in federal court for securities fraud, seeking $80,000 in damages. A pension fund lost $100 million from investing in the same fraudulent fund. This pension fund had over $12 billion in assets. The pension fund sought to intervene in the action involving the investor and the bank to assert a similar securities fraud claim against the bank. Must the pension fund's motion to intervene be granted? A. No, because the pension fund's claim pertains to a separate transaction. B. No, because although the pension fund's interest might be impaired if the individual investor's claim is litigated in its absence, the pension fund is adequately represented by the individual investor. C. Yes, because the pension fund's claim shares common questions of law and fact with the investor's claim against the bank. D. Yes, because the pension fund's interest might be impaired if the individual investor's claim is litigated in its absence and the pension fund is not adequately represented by the individual investor.

D

An inventor sued a rival in federal court for violating the inventor's patent. At a deposition attended by lawyers for both parties, the rival gave convincing testimony that his product was completely different from that of the inventor. Just before trial, the rival began serving a four-year sentence in the state penitentiary. The rival's lawyer considered the inventor's suit to be a nuisance action, and was certain that the rival's deposition testimony was all that she would need to introduce at trial to win the case. The inventor's lawyer objects to having the deposition read into evidence, arguing that a party's deposition may not be used in this way. Will the court allow the deposition to be used? A. No, because imprisonment does not make a party "unavailable." B. No, because the rival is a party. C. Yes, because the deposition of a party may be used for any purpose. D. Yes, because the rival is unavailable.

D

At a cocktail party, the host asked a guest, in private, "You are a lawyer; can you tell me whether I can be held liable for any car accidents that my guests are in as they leave my party since I secretly spiked the punch with vodka?" The guest responded, "Although I am a lawyer licensed to practice in this state, I am insufficiently familiar with the law on that topic to advise you." As it turns out, a couple was involved in a fatal accident as they were leaving the party. The surviving son of the couple initiated a wrongful death action against the host of the party. During a deposition of the guest, the attorney for the surviving son asked the guest, "You had a brief conversation with the host at the party. What did you talk about?" The attorney for the host objected to the question on the ground that answering the question would violate the attorney-client privilege of the host. Is this a valid invocation of the attorney-client privilege? A. No; the attorney-client privilege does not apply because of the crime-fraud exception. B. No; the attorney-client privilege does not apply because the guest and the host never established a formal attorney-client relationship and the guest declined to give the host any legal advice. C. Yes; the attorney-client privilege applies because the host's attorney can treat the guest as an associate, thereby extending the scope of the attorney-client privilege to the host's interactions with the guest D. Yes; the attorney-client privilege applies because the host's conversation with the guest was in confidence and sought legal advice from a licensed attorney.

D

For the past 30 years, a motorist purchased a new model of a particular automobile every two years. In the past year, the model was restyled by its manufacturer. Although the motorist still liked the automobile for other reasons, the motorist believed that the automobile was now so ugly that he did not want to be seen in one, and he would be forced to buy a different brand. The motorist was so angry that he sued the manufacturer in federal court for damages to compensate him for having to drive something other than that model car. As an attorney for the manufacturer, which of the following documents should you file first? A. An answer denying the allegations in the motorist's complaint. B. A motion for summary judgment. C. A counterclaim for malicious prosecution. D. A motion to dismiss for failure to state a claim upon which relief may be granted.

D

On February 1 a cardholder from State A initiated an action in State B state court against a credit card company incorporated and headquartered in State B for violation of State B consumer protection law. Although the complaint initially sought $50,000, on March 1 the cardholder amended the complaint to seek $80,000. The credit card company responded by filing a notice of removal in State B federal court on March 23. On April 1 of that same year, the cardholder dismissed the action and refiled it in State B state court, seeking $70,000. May the cardholder do this? A. No, because the filing of the notice of removal prevented the cardholder from dismissing the action without court approval or the consent of the credit card company. B. No, because the dismissal of the action in federal court operates as a resolution on the merits that prevents refiling the same claim in state or federal court. C. Yes, because the refiled claim is for less than the amount that would qualify for diversity jurisdiction in federal court. D. Yes, because this is the first dismissal and the cardholder had yet to file an answer or motion for summary judgment.

D

On October 15, 2017 a jogger filed a negligence action against a golfer in federal court in State A based on a July 2017 incident in which a ball hit by the golfer struck the jogger in the head. In response to the complaint, on November 1 the golfer filed and served a motion to dismiss for failure to state a claim, attaching a copy of an affidavit of a witness who claims that the golfer was out of the country on the date the incident was alleged to have occurred. The court did not exclude the affidavit and gave the jogger 14 days to file a response. The next day, the jogger filed an amended complaint changing the date of the alleged incident to June 15, 2017. The golfer objected to the plaintiff's amended complaint and moved to strike it as impermissible. May the jogger amend her complaint in this manner? A. No, because once the golfer filed a motion to dismiss accompanied by an affidavit that was not excluded, it became a summary judgment motion, which prevented the plaintiff from amending it without the court's permission. B. No, because revising the complaint in this way will expose the jogger to sanctions under Rule 11. C. Yes, because the golfer has yet to file an answer. D. Yes, because this is the jogger's first amendment and it is being made only one day after service of the golfer's motion to dismiss.

D

On September 1 a driver initiated an action in federal court against a pedestrian for recklessness that caused the driver to swerve and collide with a building. The driver sought damages for the cost of replacing the car, which was an expensive high-end luxury vehicle. On November 1 of that same year, which was after the statute of limitations period expired, the driver developed a piercing pain in her back. After visiting the doctor, the driver learned that she had suffered severe injury to her spine that would require immediate surgery. As a result, the driver sought to amend her complaint to seek additional damages for the back injury. The pedestrian opposes the amendment on the ground that it would add damages claims that would be untimely due to the expiration of the statute of limitations period. Will the driver's amended complaint relate back to the time of filing? A. no, bc the driver failed to exercise diligence in ensuring that all related claims were asserted prior to the expiration of the statute of limitations period B. no, bc the driver's failure to include person injury damages in the initial complaint was not due to mistake C. yes, bc the amendment is being offered w/in 90 days of the filing of the complaint D. yes, bc the new damages arise out of the conduct that was set out in the original complaint

D

The General Counsel of a company prepared an assessment of the potential total exposure from all pending lawsuits against the company and the likelihood of success in each case. The General Counsel did this in a spreadsheet that included the name of the litigation matter, the amount in controversy, and the likelihood that the plaintiff in each case would prevail. The General Counsel then used this information to establish the size of the company's litigation reserve. As it turns out, the ultimate losses the company faced in the litigation reflected in the General Counsel's litigation assessment was ten times the amount estimated by the General Counsel. As a result, the company had to sell off a substantial number of assets to pay the judgments. The stock price of the company suffered as a result. A shareholder initiated a securities fraud action against the company based on the company's maintenance of an inadequate litigation reserve. During discovery, the shareholder submitted a request for "all litigation assessments prepared by or for the company regarding pending litigation that formed the basis for the determination of the company's litigation reserve." If the company ultimately moves for a protective order to prevent the disclosure of this spreadsheet in response to this request (after meeting and conferring with the shareholder), how is the court likely to rule? A. Deny the motion, because this information is not protected by the attorney-client privilege. B. Deny the motion, because this information is related to the shareholder's claim in this action. C. Grant the motion, because this information is protected by the attorney-client privilege. D. Grant the motion, because this information is protected by the work product doctrine.

D

You represent a manufacturer that has just received a complaint in a federal action seeking payment for steel delivered by a steel company. The manufacturer informs you that it has already fully paid the steel company (a fact that would absolve the manufacturer of liability under the relevant law) and that the steel at issue in the case was substandard, causing the manufacturer to have to acquire additional steel from another supplier. The manufacturer asks you how it should respond to the complaint given these facts. Which of the following reflects the best advice that you could give? A. file a motion to dismiss for failure to state a claim in light of the fact that the steel company has already received full payment B. file a motion for judgment on the pleadings, attaching a copy of the receipt for payment given to the manufacturer by the steel company C. quickly initiate a separate action against the steel company for breach of contract based on the substandard quality of the steel that was supplied, seeking damages to compensate for the additional costs incurred by the manufacturer D. respond to the complaint w/ an answer containing a counterclaim for breach of contract based on the supply of substandard steel and assert payment as an affirmative defense to the steel company's claim

D

On February 1 two homeowners filed a complaint in federal court alleging that a contractor negligently constructed their home, thereby causing them injuries. On February 15 of the same year the contractor filed and served a motion to dismiss for failure to state a claim, arguing that under the applicable state law the contractor is immune from all suits alleging negligence in the construction of homes. In response, on March 15 of that year the homeowners sought to amend their complaint to allege that the contractor was reckless in constructing their home, a claim to which the contractor would not be immune. The contractor objects to the amendment. Should the amendment be allowed? A. no, bc the homeowners seek to amend their complaint more than 21 days after the filing of their original complaint B. no bc the contractor has not consented to the amendment C. no bc the contractor would be prejudiced by the amendment D. yes, bc amendments must be freely granted and there is no indication of bad faith or unfair prejudice E. yes bc this is the homeowners first amendment

D After 21 days, the court's permission is required to amend, which must be granted freely in the interest of justice. Here, there are no facts indicating that there is bad faith on the part of the homeowner and the contractor will have ample time to respond to the revised allegations, meaning there is no unfair prejudice. Thus, the court should allow the amendment. Fed. R. Civ. P. 15(a)(2).

A cellphone company initiated an action against a competitor alleging patent infringement based on the competitor's production of a phone that had multiple design components that appeared to reflect unlicensed uses of the cellphone company's patents. During discovery, the cellphone company requested detailed design plans of the competitor's allegedly infringing product, information that is critical to the cellphone company's ability to establish its claim. The competitor is concerned that such information consists of confidential trade secrets. You represent the competitor. Which of the following responses is the most likely to be successful in addressing the competitor's concern? A. Object to the request on the ground that it is overly broad and seeks information that is not necessary to proving the cellphone company's claim. B. Decline to produce the information on the ground that it is privileged and report it on a privilege log. C. Seek a protective order permitting the competitor to withhold the trade secrets on the ground that the commercial harm associated with disclosing the trade secret outweighs the benefit to the litigation, making its production unduly burdensome. D. Seek a protective order requiring that the trade secrets only be revealed to the judge and an attorney for the cellphone company.

D It recognizes that the trade secrets will have to be produced but also outlines a way that a protective order could be fashioned to protect the competitor against being harmed by the disclosure. If only the attorneys for the cellphone company see the material and not representatives of the cellphone company, that will ensure that the company is not able to improperly use the information for its commercial value. Fed. R. Civ. P. 26(c)(1)(G).

An employee initiated an action in federal court against an employer for discrimination in failing to promote the employee to an available position. During discovery, the employee requested production of company organization charts, accounting audits of the employee's department, personnel and payroll information for the duration of the employee's employment, agendas from board meetings, and work performance and payroll information for the plaintiff-employee and other employees. The request asked that these documents be produced in their native format, which means the format in which the document was created. The employer responded by producing all of the requested documents in Portable Document Format (PDF), even though that was not their native format and even though the employer ordinarily maintained all of the documents in their respective native formats. The employee objected to this form of production and after meeting and conferring with the employer, the employee filed a motion to compel production of the documents in their native format, which would consist of word processing and spreadsheet formats that would reveal information about the author, creation date, and history of each document. How is the court likely to rule on the motion to compel? A. Deny the motion, because the additional information provided in the native format versions of the documents is outside the scope of discovery. B. Deny the motion, because producing such electronically stored information in its native format would be unduly burdensome. C. Grant the motion, because electronically stored information must be produced in is native format. D. Grant the motion, because the employee is entitled to specify the form for producing electronically stored information and producing it in its native format would not be unduly burdensome.

D The Federal Rules require parties to produce documents as they are ordinarily kept in business and do permit the requesting party to specify the form of production for ESI, Fed. R. Civ. P. 34(b)(2)(E); the responding party must comply with that request unless it is unduly burdensome, which it is not here. Further, metadata is fully discoverable and should be produced when information is requested in its native format. Williams v. Sprint/United Management Co., 230 F.R.D. 640, 652 (D. Kan. 2005) ("[W]hen a party is ordered to produce electronic documents as they are maintained in the ordinary course of business, the producing party should produce the electronic documents with their metadata intact, unless that party timely objects to production of metadata, the parties agree that the metadata should not be produced, or the producing party requests a protective order.").

An investor initiated a securities fraud action against a securities broker in federal court. To draft the complaint, the attorney for the investor learned the names of the defendants by asking the investor to recall them from memory. Upon receiving the complaint, the broker's attorney determined that the broker was not involved in the transaction in question at all; instead, the proper party was another broker with a similar but different name. The broker's attorney thus drafted a motion for sanctions under Rule 11 and served it on the investor. Thirty days later, the investor withdrew her pleading and filed an amended complaint that no longer sought any relief against the broker. In response, the broker's attorney filed the previously served motion for sanctions under Rule 11. After permitting the investor an opportunity to respond in opposition to the motion, the court granted the motion for Rule 11 sanctions. Was the court's grant proper? A. No, because sanctions may not be entered against a party who is represented by an attorney. B. No, because there is no indication that the investor filed the complaint in bad faith. C. No, because the investor withdrew the offending pleading prior to the filing of the motion for sanctions. D. Yes, because naming the broker needlessly increased the cost of this litigation. E. Yes, because the investor's attorney failed to conduct a reasonable prefiling inquiry and the naming of the broker as a defendant lacked any evidentiary support.

E . The facts show that the attorney did not conduct a reasonable prefiling inquiry and that the allegation of the broker's involvement lacked any factual foundation, violations of Rule 11(b). The fact that the complaint has been withdrawn does not preclude a finding of a Rule 11 violation and the imposition of sanctions because the withdrawal came after the 21-day safe harbor period had elapsed. Fed. R. Civ. P. 11(c)(2). A court might still find sanctions appropriate to deter a repetition of such errors by the attorney. The party seeking sanctions may also wish to pursue attorney's fees to compensate for the cost of responding to the offending complaint and of filing the motion for sanctions. Fed. R. Civ. P. 11(c)(4).


Ensembles d'études connexes

PSYCH OF LEARNING CHAPTER 3, PSYCH of Learning Chapter 4, PSYCH of Learning Chapter 5

View Set

Health Edu HS Canvas CCSD Module 3 Quiz: Stress, Social and Emotional Health, Suicide Prevention, and Mental Illness

View Set

Criminal Law & Procedure Collected from Others

View Set